Prove that n squared is less than or equal to 2 to the n by induction. [duplicate] Announcing the arrival of Valued Associate #679: Cesar Manara Planned maintenance scheduled April 17/18, 2019 at 00:00UTC (8:00pm US/Eastern)Proof of $n^2 leq 2^n$.Proof that $n^2 < 2^n$Prove that $n^k < 2^n$ for all large enough $n$Induction proof $n^2 < 2^n$ for $n > 4$Prove by induction that $2^n > n^2$Proof by induction: $ 2^n ge n^2$ for $nge4$Please prove that $nchoose r < (n+1)^r$. My induction-based proof is ugly!Proof of $n^2 leq 2^n$.Prove that $n^2 > n+1 quadforall n geq 2$ using mathematical inductionprove inequality by induction — Discrete mathProve: $ 1times3 +2times4 + cdots + n(n+2) = frac16 times n(n+1)(2n+7)$ using InductionProve by induction that $a_n+1 = a_n + a_n$Induction proof that a square of a sum is less than a halfProve Big-Oh using InductionInduction Proof: Prove that if $|N(w)| = 3^k$ for $kgeq 0$, then $N(w) = (-3)^k$.Given $x_1 := sqrt2$ and $x_n+1 :=sqrt2x_n $, prove $sqrt2 ≤ x_n ≤ 2$

Why am I getting the error "non-boolean type specified in a context where a condition is expected" for this request?

2001: A Space Odyssey's use of the song "Daisy Bell" (Bicycle Built for Two); life imitates art or vice-versa?

Extract all GPU name, model and GPU ram

When were vectors invented?

Error "illegal generic type for instanceof" when using local classes

Storing hydrofluoric acid before the invention of plastics

porting install scripts : can rpm replace apt?

Is it fair for a professor to grade us on the possession of past papers?

How does the particle を relate to the verb 行く in the structure「A を + B に行く」?

Apollo command module space walk?

How do I keep my slimes from escaping their pens?

If a contract sometimes uses the wrong name, is it still valid?

Why did the Falcon Heavy center core fall off the ASDS OCISLY barge?

What does the "x" in "x86" represent?

What exactly is a "Meth" in Altered Carbon?

Why was the term "discrete" used in discrete logarithm?

Do I really need recursive chmod to restrict access to a folder?

What is Arya's weapon design?

How to tell that you are a giant?

How to deal with a team lead who never gives me credit?

Coloring maths inside a tcolorbox

Why is "Consequences inflicted." not a sentence?

What is the meaning of the new sigil in Game of Thrones Season 8 intro?

ListPlot join points by nearest neighbor rather than order



Prove that n squared is less than or equal to 2 to the n by induction. [duplicate]



Announcing the arrival of Valued Associate #679: Cesar Manara
Planned maintenance scheduled April 17/18, 2019 at 00:00UTC (8:00pm US/Eastern)Proof of $n^2 leq 2^n$.Proof that $n^2 < 2^n$Prove that $n^k < 2^n$ for all large enough $n$Induction proof $n^2 < 2^n$ for $n > 4$Prove by induction that $2^n > n^2$Proof by induction: $ 2^n ge n^2$ for $nge4$Please prove that $nchoose r < (n+1)^r$. My induction-based proof is ugly!Proof of $n^2 leq 2^n$.Prove that $n^2 > n+1 quadforall n geq 2$ using mathematical inductionprove inequality by induction — Discrete mathProve: $ 1times3 +2times4 + cdots + n(n+2) = frac16 times n(n+1)(2n+7)$ using InductionProve by induction that $a_n+1 = a_n + a_n$Induction proof that a square of a sum is less than a halfProve Big-Oh using InductionInduction Proof: Prove that if $|N(w)| = 3^k$ for $kgeq 0$, then $N(w) = (-3)^k$.Given $x_1 := sqrt2$ and $x_n+1 :=sqrt2x_n $, prove $sqrt2 ≤ x_n ≤ 2$










1












$begingroup$



This question already has an answer here:



  • Proof of $n^2 leq 2^n$.

    4 answers



I've been asked to prove by induction that $n^2leq 2^n$, and told it is true $ forall nin mathbbN,n>3$



I think I have found the right way to the proof, but I'm not sure since I get stuck half-way there. What I did was taking a base case of $n=4$ and tested it, and it resulted to be true. Then I assumed it would be true for some number $k$, such that $n=k$ and $k^2leq 2^k$, and attempted to prove



$(k+1)^2 leq 2^k+1$



And this is how I attempted to prove this. First of all, I started with my assumption.



$=k^2leq 2^k$



$=2k^2leq2^k+1$



Then I tried to prove that $(k+1)^2 leq 2k^2$, for this would imply my thesis, i.e. $(k+1)^2leq2^k+1$. So I went forth on my effort:



$(k+1)^2≤2k^2$



$=k^2+2k+1leq2k^2$



$=2k+1leq k^2$



(By assumption)



$=2k+1leq 2^k$



Now that I simplified it, I need to prove this is true; this is, prove that $(k+1)^2 leq 2k^2$. So I take a base case of $n=4$ and in deed it satisfies the inequality. So I assume it is true for some number $j, k=j$ and try to prove it. Nevertheless I have failed in trying to prove this, I don't really know if my steps so far are right or wrong. Is my reasoning okay? And if its, how can I prove $2k+1leq 2^k$? Thank you in advance.










share|cite|improve this question











$endgroup$



marked as duplicate by dantopa, Cesareo, Lee David Chung Lin, Eevee Trainer, Leucippus Mar 27 at 4:57


This question has been asked before and already has an answer. If those answers do not fully address your question, please ask a new question.

















  • $begingroup$
    This question has been asked so many times already. Please make an attempt to search for the problem. math.stackexchange.com/questions/319913/proof-that-n2-2n/…, math.stackexchange.com/questions/263825/…, math.stackexchange.com/questions/263825/…
    $endgroup$
    – JavaMan
    Mar 26 at 17:30











  • $begingroup$
    math.stackexchange.com/questions/876127/…, math.stackexchange.com/questions/1489889/…, math.stackexchange.com/questions/439026/…
    $endgroup$
    – JavaMan
    Mar 26 at 17:30










  • $begingroup$
    When you realize that you must prove $2k + 1<k^2$ !!!! DONT !!! go back to your assumption to prove $2k+1 < 2^k$. For one thing proving $2k + 1< 2^k$ won't prove $k^2 < 2k + 1 < 2^k$ is impossible, but more importantly, you got here via the assumption. Going back will be circular. You have gotten to the "practical world" where proving $2k+1< k^2$ is your direct way of proving your proposition. So prove it directly. Prove $2k + 1 < k^2$. That should be easy.
    $endgroup$
    – fleablood
    Mar 26 at 17:37











  • $begingroup$
    ... for one thing $k^2 -2k - 1= k^2 - 2k + 1 - 2=(k-1)^2 - 2ge 3^2 -2 = 7 > 0$ so $k^2 > 2k + 1$.
    $endgroup$
    – fleablood
    Mar 26 at 17:43















1












$begingroup$



This question already has an answer here:



  • Proof of $n^2 leq 2^n$.

    4 answers



I've been asked to prove by induction that $n^2leq 2^n$, and told it is true $ forall nin mathbbN,n>3$



I think I have found the right way to the proof, but I'm not sure since I get stuck half-way there. What I did was taking a base case of $n=4$ and tested it, and it resulted to be true. Then I assumed it would be true for some number $k$, such that $n=k$ and $k^2leq 2^k$, and attempted to prove



$(k+1)^2 leq 2^k+1$



And this is how I attempted to prove this. First of all, I started with my assumption.



$=k^2leq 2^k$



$=2k^2leq2^k+1$



Then I tried to prove that $(k+1)^2 leq 2k^2$, for this would imply my thesis, i.e. $(k+1)^2leq2^k+1$. So I went forth on my effort:



$(k+1)^2≤2k^2$



$=k^2+2k+1leq2k^2$



$=2k+1leq k^2$



(By assumption)



$=2k+1leq 2^k$



Now that I simplified it, I need to prove this is true; this is, prove that $(k+1)^2 leq 2k^2$. So I take a base case of $n=4$ and in deed it satisfies the inequality. So I assume it is true for some number $j, k=j$ and try to prove it. Nevertheless I have failed in trying to prove this, I don't really know if my steps so far are right or wrong. Is my reasoning okay? And if its, how can I prove $2k+1leq 2^k$? Thank you in advance.










share|cite|improve this question











$endgroup$



marked as duplicate by dantopa, Cesareo, Lee David Chung Lin, Eevee Trainer, Leucippus Mar 27 at 4:57


This question has been asked before and already has an answer. If those answers do not fully address your question, please ask a new question.

















  • $begingroup$
    This question has been asked so many times already. Please make an attempt to search for the problem. math.stackexchange.com/questions/319913/proof-that-n2-2n/…, math.stackexchange.com/questions/263825/…, math.stackexchange.com/questions/263825/…
    $endgroup$
    – JavaMan
    Mar 26 at 17:30











  • $begingroup$
    math.stackexchange.com/questions/876127/…, math.stackexchange.com/questions/1489889/…, math.stackexchange.com/questions/439026/…
    $endgroup$
    – JavaMan
    Mar 26 at 17:30










  • $begingroup$
    When you realize that you must prove $2k + 1<k^2$ !!!! DONT !!! go back to your assumption to prove $2k+1 < 2^k$. For one thing proving $2k + 1< 2^k$ won't prove $k^2 < 2k + 1 < 2^k$ is impossible, but more importantly, you got here via the assumption. Going back will be circular. You have gotten to the "practical world" where proving $2k+1< k^2$ is your direct way of proving your proposition. So prove it directly. Prove $2k + 1 < k^2$. That should be easy.
    $endgroup$
    – fleablood
    Mar 26 at 17:37











  • $begingroup$
    ... for one thing $k^2 -2k - 1= k^2 - 2k + 1 - 2=(k-1)^2 - 2ge 3^2 -2 = 7 > 0$ so $k^2 > 2k + 1$.
    $endgroup$
    – fleablood
    Mar 26 at 17:43













1












1








1





$begingroup$



This question already has an answer here:



  • Proof of $n^2 leq 2^n$.

    4 answers



I've been asked to prove by induction that $n^2leq 2^n$, and told it is true $ forall nin mathbbN,n>3$



I think I have found the right way to the proof, but I'm not sure since I get stuck half-way there. What I did was taking a base case of $n=4$ and tested it, and it resulted to be true. Then I assumed it would be true for some number $k$, such that $n=k$ and $k^2leq 2^k$, and attempted to prove



$(k+1)^2 leq 2^k+1$



And this is how I attempted to prove this. First of all, I started with my assumption.



$=k^2leq 2^k$



$=2k^2leq2^k+1$



Then I tried to prove that $(k+1)^2 leq 2k^2$, for this would imply my thesis, i.e. $(k+1)^2leq2^k+1$. So I went forth on my effort:



$(k+1)^2≤2k^2$



$=k^2+2k+1leq2k^2$



$=2k+1leq k^2$



(By assumption)



$=2k+1leq 2^k$



Now that I simplified it, I need to prove this is true; this is, prove that $(k+1)^2 leq 2k^2$. So I take a base case of $n=4$ and in deed it satisfies the inequality. So I assume it is true for some number $j, k=j$ and try to prove it. Nevertheless I have failed in trying to prove this, I don't really know if my steps so far are right or wrong. Is my reasoning okay? And if its, how can I prove $2k+1leq 2^k$? Thank you in advance.










share|cite|improve this question











$endgroup$





This question already has an answer here:



  • Proof of $n^2 leq 2^n$.

    4 answers



I've been asked to prove by induction that $n^2leq 2^n$, and told it is true $ forall nin mathbbN,n>3$



I think I have found the right way to the proof, but I'm not sure since I get stuck half-way there. What I did was taking a base case of $n=4$ and tested it, and it resulted to be true. Then I assumed it would be true for some number $k$, such that $n=k$ and $k^2leq 2^k$, and attempted to prove



$(k+1)^2 leq 2^k+1$



And this is how I attempted to prove this. First of all, I started with my assumption.



$=k^2leq 2^k$



$=2k^2leq2^k+1$



Then I tried to prove that $(k+1)^2 leq 2k^2$, for this would imply my thesis, i.e. $(k+1)^2leq2^k+1$. So I went forth on my effort:



$(k+1)^2≤2k^2$



$=k^2+2k+1leq2k^2$



$=2k+1leq k^2$



(By assumption)



$=2k+1leq 2^k$



Now that I simplified it, I need to prove this is true; this is, prove that $(k+1)^2 leq 2k^2$. So I take a base case of $n=4$ and in deed it satisfies the inequality. So I assume it is true for some number $j, k=j$ and try to prove it. Nevertheless I have failed in trying to prove this, I don't really know if my steps so far are right or wrong. Is my reasoning okay? And if its, how can I prove $2k+1leq 2^k$? Thank you in advance.





This question already has an answer here:



  • Proof of $n^2 leq 2^n$.

    4 answers







discrete-mathematics proof-verification inequality proof-writing induction






share|cite|improve this question















share|cite|improve this question













share|cite|improve this question




share|cite|improve this question








edited Mar 26 at 17:27









Martin Hansen

975115




975115










asked Mar 26 at 17:12









AngelusSilesiusAngelusSilesius

495




495




marked as duplicate by dantopa, Cesareo, Lee David Chung Lin, Eevee Trainer, Leucippus Mar 27 at 4:57


This question has been asked before and already has an answer. If those answers do not fully address your question, please ask a new question.









marked as duplicate by dantopa, Cesareo, Lee David Chung Lin, Eevee Trainer, Leucippus Mar 27 at 4:57


This question has been asked before and already has an answer. If those answers do not fully address your question, please ask a new question.













  • $begingroup$
    This question has been asked so many times already. Please make an attempt to search for the problem. math.stackexchange.com/questions/319913/proof-that-n2-2n/…, math.stackexchange.com/questions/263825/…, math.stackexchange.com/questions/263825/…
    $endgroup$
    – JavaMan
    Mar 26 at 17:30











  • $begingroup$
    math.stackexchange.com/questions/876127/…, math.stackexchange.com/questions/1489889/…, math.stackexchange.com/questions/439026/…
    $endgroup$
    – JavaMan
    Mar 26 at 17:30










  • $begingroup$
    When you realize that you must prove $2k + 1<k^2$ !!!! DONT !!! go back to your assumption to prove $2k+1 < 2^k$. For one thing proving $2k + 1< 2^k$ won't prove $k^2 < 2k + 1 < 2^k$ is impossible, but more importantly, you got here via the assumption. Going back will be circular. You have gotten to the "practical world" where proving $2k+1< k^2$ is your direct way of proving your proposition. So prove it directly. Prove $2k + 1 < k^2$. That should be easy.
    $endgroup$
    – fleablood
    Mar 26 at 17:37











  • $begingroup$
    ... for one thing $k^2 -2k - 1= k^2 - 2k + 1 - 2=(k-1)^2 - 2ge 3^2 -2 = 7 > 0$ so $k^2 > 2k + 1$.
    $endgroup$
    – fleablood
    Mar 26 at 17:43
















  • $begingroup$
    This question has been asked so many times already. Please make an attempt to search for the problem. math.stackexchange.com/questions/319913/proof-that-n2-2n/…, math.stackexchange.com/questions/263825/…, math.stackexchange.com/questions/263825/…
    $endgroup$
    – JavaMan
    Mar 26 at 17:30











  • $begingroup$
    math.stackexchange.com/questions/876127/…, math.stackexchange.com/questions/1489889/…, math.stackexchange.com/questions/439026/…
    $endgroup$
    – JavaMan
    Mar 26 at 17:30










  • $begingroup$
    When you realize that you must prove $2k + 1<k^2$ !!!! DONT !!! go back to your assumption to prove $2k+1 < 2^k$. For one thing proving $2k + 1< 2^k$ won't prove $k^2 < 2k + 1 < 2^k$ is impossible, but more importantly, you got here via the assumption. Going back will be circular. You have gotten to the "practical world" where proving $2k+1< k^2$ is your direct way of proving your proposition. So prove it directly. Prove $2k + 1 < k^2$. That should be easy.
    $endgroup$
    – fleablood
    Mar 26 at 17:37











  • $begingroup$
    ... for one thing $k^2 -2k - 1= k^2 - 2k + 1 - 2=(k-1)^2 - 2ge 3^2 -2 = 7 > 0$ so $k^2 > 2k + 1$.
    $endgroup$
    – fleablood
    Mar 26 at 17:43















$begingroup$
This question has been asked so many times already. Please make an attempt to search for the problem. math.stackexchange.com/questions/319913/proof-that-n2-2n/…, math.stackexchange.com/questions/263825/…, math.stackexchange.com/questions/263825/…
$endgroup$
– JavaMan
Mar 26 at 17:30





$begingroup$
This question has been asked so many times already. Please make an attempt to search for the problem. math.stackexchange.com/questions/319913/proof-that-n2-2n/…, math.stackexchange.com/questions/263825/…, math.stackexchange.com/questions/263825/…
$endgroup$
– JavaMan
Mar 26 at 17:30













$begingroup$
math.stackexchange.com/questions/876127/…, math.stackexchange.com/questions/1489889/…, math.stackexchange.com/questions/439026/…
$endgroup$
– JavaMan
Mar 26 at 17:30




$begingroup$
math.stackexchange.com/questions/876127/…, math.stackexchange.com/questions/1489889/…, math.stackexchange.com/questions/439026/…
$endgroup$
– JavaMan
Mar 26 at 17:30












$begingroup$
When you realize that you must prove $2k + 1<k^2$ !!!! DONT !!! go back to your assumption to prove $2k+1 < 2^k$. For one thing proving $2k + 1< 2^k$ won't prove $k^2 < 2k + 1 < 2^k$ is impossible, but more importantly, you got here via the assumption. Going back will be circular. You have gotten to the "practical world" where proving $2k+1< k^2$ is your direct way of proving your proposition. So prove it directly. Prove $2k + 1 < k^2$. That should be easy.
$endgroup$
– fleablood
Mar 26 at 17:37





$begingroup$
When you realize that you must prove $2k + 1<k^2$ !!!! DONT !!! go back to your assumption to prove $2k+1 < 2^k$. For one thing proving $2k + 1< 2^k$ won't prove $k^2 < 2k + 1 < 2^k$ is impossible, but more importantly, you got here via the assumption. Going back will be circular. You have gotten to the "practical world" where proving $2k+1< k^2$ is your direct way of proving your proposition. So prove it directly. Prove $2k + 1 < k^2$. That should be easy.
$endgroup$
– fleablood
Mar 26 at 17:37













$begingroup$
... for one thing $k^2 -2k - 1= k^2 - 2k + 1 - 2=(k-1)^2 - 2ge 3^2 -2 = 7 > 0$ so $k^2 > 2k + 1$.
$endgroup$
– fleablood
Mar 26 at 17:43




$begingroup$
... for one thing $k^2 -2k - 1= k^2 - 2k + 1 - 2=(k-1)^2 - 2ge 3^2 -2 = 7 > 0$ so $k^2 > 2k + 1$.
$endgroup$
– fleablood
Mar 26 at 17:43










1 Answer
1






active

oldest

votes


















0












$begingroup$

You need to prove $2k + 1 le k^2$.



Do it this way: $1 < k$ so $2k + 1 < 2k + k = 3k$. And $3 < k$ so $3k < k^2$.



So to put it together:



Induction step:



If $k^2 < 2^k; k > 3$ then



$(k+1)^2 = k^2 + 2k + 1 < $



$k^2 + 2k + k = k^2 + 3k < $



$k^2 + k*k = 2k^2 < $



$2*2^k = 2^k+1$.



.....



.... or simply note...



$2k + 1 < k^2 iff$



$1 < k^2 - 2k iff$



$2 < k^2 - 2k + 1 = (k-1)^2$.



And $k-1 ge 3$ the $(k-1)^2 ge 9 > 2$.






share|cite|improve this answer











$endgroup$












  • $begingroup$
    Such a neat reasoning, I feel silly I couldn't do this myself, haha. Excelent, @fleablood, I appreciate the fact that you took the trouble to answer my question. Thank you!
    $endgroup$
    – AngelusSilesius
    Mar 26 at 19:30










  • $begingroup$
    The last line in the first part should probably read $2*2^k colorred= 2^k+1.$
    $endgroup$
    – CiaPan
    Mar 26 at 20:36










  • $begingroup$
    ooppps.........
    $endgroup$
    – fleablood
    Mar 26 at 21:08

















1 Answer
1






active

oldest

votes








1 Answer
1






active

oldest

votes









active

oldest

votes






active

oldest

votes









0












$begingroup$

You need to prove $2k + 1 le k^2$.



Do it this way: $1 < k$ so $2k + 1 < 2k + k = 3k$. And $3 < k$ so $3k < k^2$.



So to put it together:



Induction step:



If $k^2 < 2^k; k > 3$ then



$(k+1)^2 = k^2 + 2k + 1 < $



$k^2 + 2k + k = k^2 + 3k < $



$k^2 + k*k = 2k^2 < $



$2*2^k = 2^k+1$.



.....



.... or simply note...



$2k + 1 < k^2 iff$



$1 < k^2 - 2k iff$



$2 < k^2 - 2k + 1 = (k-1)^2$.



And $k-1 ge 3$ the $(k-1)^2 ge 9 > 2$.






share|cite|improve this answer











$endgroup$












  • $begingroup$
    Such a neat reasoning, I feel silly I couldn't do this myself, haha. Excelent, @fleablood, I appreciate the fact that you took the trouble to answer my question. Thank you!
    $endgroup$
    – AngelusSilesius
    Mar 26 at 19:30










  • $begingroup$
    The last line in the first part should probably read $2*2^k colorred= 2^k+1.$
    $endgroup$
    – CiaPan
    Mar 26 at 20:36










  • $begingroup$
    ooppps.........
    $endgroup$
    – fleablood
    Mar 26 at 21:08















0












$begingroup$

You need to prove $2k + 1 le k^2$.



Do it this way: $1 < k$ so $2k + 1 < 2k + k = 3k$. And $3 < k$ so $3k < k^2$.



So to put it together:



Induction step:



If $k^2 < 2^k; k > 3$ then



$(k+1)^2 = k^2 + 2k + 1 < $



$k^2 + 2k + k = k^2 + 3k < $



$k^2 + k*k = 2k^2 < $



$2*2^k = 2^k+1$.



.....



.... or simply note...



$2k + 1 < k^2 iff$



$1 < k^2 - 2k iff$



$2 < k^2 - 2k + 1 = (k-1)^2$.



And $k-1 ge 3$ the $(k-1)^2 ge 9 > 2$.






share|cite|improve this answer











$endgroup$












  • $begingroup$
    Such a neat reasoning, I feel silly I couldn't do this myself, haha. Excelent, @fleablood, I appreciate the fact that you took the trouble to answer my question. Thank you!
    $endgroup$
    – AngelusSilesius
    Mar 26 at 19:30










  • $begingroup$
    The last line in the first part should probably read $2*2^k colorred= 2^k+1.$
    $endgroup$
    – CiaPan
    Mar 26 at 20:36










  • $begingroup$
    ooppps.........
    $endgroup$
    – fleablood
    Mar 26 at 21:08













0












0








0





$begingroup$

You need to prove $2k + 1 le k^2$.



Do it this way: $1 < k$ so $2k + 1 < 2k + k = 3k$. And $3 < k$ so $3k < k^2$.



So to put it together:



Induction step:



If $k^2 < 2^k; k > 3$ then



$(k+1)^2 = k^2 + 2k + 1 < $



$k^2 + 2k + k = k^2 + 3k < $



$k^2 + k*k = 2k^2 < $



$2*2^k = 2^k+1$.



.....



.... or simply note...



$2k + 1 < k^2 iff$



$1 < k^2 - 2k iff$



$2 < k^2 - 2k + 1 = (k-1)^2$.



And $k-1 ge 3$ the $(k-1)^2 ge 9 > 2$.






share|cite|improve this answer











$endgroup$



You need to prove $2k + 1 le k^2$.



Do it this way: $1 < k$ so $2k + 1 < 2k + k = 3k$. And $3 < k$ so $3k < k^2$.



So to put it together:



Induction step:



If $k^2 < 2^k; k > 3$ then



$(k+1)^2 = k^2 + 2k + 1 < $



$k^2 + 2k + k = k^2 + 3k < $



$k^2 + k*k = 2k^2 < $



$2*2^k = 2^k+1$.



.....



.... or simply note...



$2k + 1 < k^2 iff$



$1 < k^2 - 2k iff$



$2 < k^2 - 2k + 1 = (k-1)^2$.



And $k-1 ge 3$ the $(k-1)^2 ge 9 > 2$.







share|cite|improve this answer














share|cite|improve this answer



share|cite|improve this answer








edited Mar 26 at 21:09

























answered Mar 26 at 17:29









fleabloodfleablood

1




1











  • $begingroup$
    Such a neat reasoning, I feel silly I couldn't do this myself, haha. Excelent, @fleablood, I appreciate the fact that you took the trouble to answer my question. Thank you!
    $endgroup$
    – AngelusSilesius
    Mar 26 at 19:30










  • $begingroup$
    The last line in the first part should probably read $2*2^k colorred= 2^k+1.$
    $endgroup$
    – CiaPan
    Mar 26 at 20:36










  • $begingroup$
    ooppps.........
    $endgroup$
    – fleablood
    Mar 26 at 21:08
















  • $begingroup$
    Such a neat reasoning, I feel silly I couldn't do this myself, haha. Excelent, @fleablood, I appreciate the fact that you took the trouble to answer my question. Thank you!
    $endgroup$
    – AngelusSilesius
    Mar 26 at 19:30










  • $begingroup$
    The last line in the first part should probably read $2*2^k colorred= 2^k+1.$
    $endgroup$
    – CiaPan
    Mar 26 at 20:36










  • $begingroup$
    ooppps.........
    $endgroup$
    – fleablood
    Mar 26 at 21:08















$begingroup$
Such a neat reasoning, I feel silly I couldn't do this myself, haha. Excelent, @fleablood, I appreciate the fact that you took the trouble to answer my question. Thank you!
$endgroup$
– AngelusSilesius
Mar 26 at 19:30




$begingroup$
Such a neat reasoning, I feel silly I couldn't do this myself, haha. Excelent, @fleablood, I appreciate the fact that you took the trouble to answer my question. Thank you!
$endgroup$
– AngelusSilesius
Mar 26 at 19:30












$begingroup$
The last line in the first part should probably read $2*2^k colorred= 2^k+1.$
$endgroup$
– CiaPan
Mar 26 at 20:36




$begingroup$
The last line in the first part should probably read $2*2^k colorred= 2^k+1.$
$endgroup$
– CiaPan
Mar 26 at 20:36












$begingroup$
ooppps.........
$endgroup$
– fleablood
Mar 26 at 21:08




$begingroup$
ooppps.........
$endgroup$
– fleablood
Mar 26 at 21:08



Popular posts from this blog

Lowndes Grove History Architecture References Navigation menu32°48′6″N 79°57′58″W / 32.80167°N 79.96611°W / 32.80167; -79.9661132°48′6″N 79°57′58″W / 32.80167°N 79.96611°W / 32.80167; -79.9661178002500"National Register Information System"Historic houses of South Carolina"Lowndes Grove""+32° 48' 6.00", −79° 57' 58.00""Lowndes Grove, Charleston County (260 St. Margaret St., Charleston)""Lowndes Grove"The Charleston ExpositionIt Happened in South Carolina"Lowndes Grove (House), Saint Margaret Street & Sixth Avenue, Charleston, Charleston County, SC(Photographs)"Plantations of the Carolina Low Countrye

random experiment with two different functions on unit interval Announcing the arrival of Valued Associate #679: Cesar Manara Planned maintenance scheduled April 23, 2019 at 00:00UTC (8:00pm US/Eastern)Random variable and probability space notionsRandom Walk with EdgesFinding functions where the increase over a random interval is Poisson distributedNumber of days until dayCan an observed event in fact be of zero probability?Unit random processmodels of coins and uniform distributionHow to get the number of successes given $n$ trials , probability $P$ and a random variable $X$Absorbing Markov chain in a computer. Is “almost every” turned into always convergence in computer executions?Stopped random walk is not uniformly integrable

How should I support this large drywall patch? Planned maintenance scheduled April 23, 2019 at 00:00UTC (8:00pm US/Eastern) Announcing the arrival of Valued Associate #679: Cesar Manara Unicorn Meta Zoo #1: Why another podcast?How do I cover large gaps in drywall?How do I keep drywall around a patch from crumbling?Can I glue a second layer of drywall?How to patch long strip on drywall?Large drywall patch: how to avoid bulging seams?Drywall Mesh Patch vs. Bulge? To remove or not to remove?How to fix this drywall job?Prep drywall before backsplashWhat's the best way to fix this horrible drywall patch job?Drywall patching using 3M Patch Plus Primer